User Avatar
51891
Joined
Apr 2025
Subscription
Free

Hi guys,

I want to make an argument and delete LSAT 25, S2, Q10 from the problem set because the answer choice is poorly written.

Without diving too deeply what the question is saying, it is about some spider webs that emits UV and conclusion is that the insects get attracted because of the pattern.

Naturally, after reading the question, the weakest spot from the argument that I had naturally draw without reading the answer choices has to do with this specific pattern and maybe it connects with "food or mating".

And the answer choice E, which is the correct answer choice because it proves that okay insects are attracted to the UV. But that is entirely restating the premise. The answer choice is made correct because we got A, B, C irrelevant and answer choice D being 180. SO answer choice E, which doesn't do anything that sort of just rephrase the premise, being the right answer.

Please let me know how you feel. And if you feel that the answer choice certainly strengthens I be very honored to hear your reasoning because I just don't see how answer choice E is different from confirming the premise and the gap still leaves untouched.

Admin note: edited title

0

Hi guys,

Please help if you can. The question is filled with technical terms, which I know that I should just replace it. But then, the answer choices put in more of those terms and I literally felt like my brain just had blown up after doing this one single question.

But in any case, I have some questions. Please help and it may help to strengthen your ability too because the question I guess is not an easy one.

When it comes to this kind of questions, what is your approach in terms of understanding what this question is saying?

When it comes to the answer choices, with some fuzzy knowledge of what had just happened, how do you choice the correct one? For example, please take a look at answer choice C. The state of my brain was already blown up after reading and analyzing the stimulus and this term "cerebrospinal fluid" gets throw in. Naturally I picked it with the hope that JY will say in the video "we don't know that". I mean where did the stimulus say about fluids? Isn't it about some stuff attacking some other stuff.

How do you understand answer choice A?

In the end of all the elimination, I have A & C left. And I finally eliminated A, the correct answer. My brain process went like the following:

It says "Gamma interferon stops white blood cells from producing myelin-destroying compounds". But wait, white blood cell produce myelin? What is this dash doing here? What does destroying compounds mean? Isn't the problem that white blood cell is killing the myelin instead of producing it? Does Gamma stop white blood cell? All I know is that Gamma doesn't work.

In the end, I am left with an answer choice A that I could hardly understand and answer choice C with a term" spinal fluid". So I guessed for C.

Please let me know how you would approach the problem and how your mind works when you are reading this kind of passage which is filled with technical terms and your approach to answer choice A & C.

Thanks in advance.

Admin note: edited title

https://classic.7sage.com/lsat_explanations/lsat-35-section-4-question-22/

0
User Avatar
51891
Tuesday, Feb 14 2017

Will remember it in the future. Thank you very much.

1
User Avatar
51891
Tuesday, Feb 14 2017

@rahelaalam514

0
User Avatar
51891
Tuesday, Feb 14 2017

@tutordavidlevine115 said:

it's helpful to know that blocking an alternate cause can be a necessary assumption of every causation argument drawn from correlative premises.

fantastic. 太棒了

0
User Avatar
51891
Tuesday, Feb 14 2017

@51891

Your explanation is always helpful. But I am just confused about one thing. Not so much as how the problem is solved, but why is the answer an NA. I presume instead that it is a SA.

0

Hi Guys,

I am having a bit of difficulty with this question.

https://classic.7sage.com/lesson/banana-epidemics-na-question/

https://classic.7sage.com/lsat_explanations/lsat-34-section-3-question-03/

It's the banana question.

I have a hard time distinguish B. I mean, it is a sufficient assumption, yet it is the correct answer for the necessary assumption too.

I am feeling a bit, how do you call it, mumble jumble right now.

Can any one help to explain?

0

Hi guys,

I was wondering if you wondered the same thing when you entered into this lesson question:

https://classic.7sage.com/lesson/regulating-the-banks-na-question/

https://classic.7sage.com/lsat_explanations/lsat-30-section-2-question-15/

Is it correct if I translate the argument into a logic as:

Tightening-->Loan Less-->Economic Downturn

Tightening

Conclusion: Economic downturn

Aside of this logical flaw, the problem is wrong due to temporal causation flaw.

Am I correct about this?

If so, I guess you can still generate answer choice if the argument is bad as a start even logically invalid?

Thanks,

Panda

0
User Avatar
51891
Friday, Feb 10 2017

This is great input. I went into modal logic.

It's theory is one much like yours.

1

Hi all,

Just a quick question, does the word "can" indicate a relationship or is it indicate like "could", "might" a probability?

For instance, the sentence, "Some reporters can scoop all of the reporters", can you translate it into: Reporter X(-Some-)Scoop all of the reporters. Or is it just a statement indicating probability of this relationship?

if that is not true, then, as a rule of thumb, you can never translate a probabilistic statement into a conditional statement since conditional statement are 100% of occurrence?

0
User Avatar
51891
Thursday, Feb 09 2017

2 ways I have find are: we can have the necessary without the sufficient or some times the necessary condition does not lead to the sufficient

0
User Avatar

Thursday, Feb 09 2017

51891

Embedded Conditional

Hi Guys,

I was trying to prove the following statement, please help to see if it is correct:

(A-->B)-->C

not (A-->B) or C

(A some/and B) or C

C->(A some/and B)

is there more step to go below this?

So all I can get to is that, if C fails, then some A are not B.

0
User Avatar
51891
Thursday, Feb 09 2017

Your explanation are more than helpful. Thank you very much.

But just to add on. If negation of a invalid conclusion is one way to test a student's understanding for SA and NA mistake, is there other ways besides explicitly stating it out? Do you mind share them from your experience?

0
User Avatar
51891
Thursday, Feb 09 2017

@51891 said:

I don't think the application of causal reason is that helpful for 25-4-23. 25-4-23 is conditional heavy, not causal. The problem with 25-4-23 is that are given A--->B--->C in the premises and the conclusion states C---->A. Answer choice (B) gives us the negation of this conclusion. This is one of several ways in which the LSAT tests our understanding of sufficient/necessary conditions.

Edit=changing the spelling of "causal" from "casual"

@51891

0
User Avatar
51891
Wednesday, Feb 08 2017

@51891

Hi there, I agree 100% of what you had said.

But let's consider the following and ignore the question:

If A cause B cause C, then it concludes C causes A; this is wrong because the possibility of C some A, or C and A, is ignored.

If most A cause B, then B, therefore A,then, some B is not A is ignored,

Is this correct?

0
User Avatar

Wednesday, Feb 08 2017

51891

Flaw confusion, please help!!!

Hi guys,

I have some confusion going on here between the difference of the following question:

https://classic.7sage.com/lsat_explanations/lsat-25-section-4-question-23/

https://classic.7sage.com/lsat_explanations/lsat-27-section-1-question-23/

In sum,

From PT25-S4-Q23, we learned that:

If A cause B cause C, then it concludes C causes A; this is wrong because the possibility of C some A, or C and A, is ignored.

If that is true, so why is: PT27-S1-Q23 answer choice A wrong.

If most A cause B, then B, therefore A, the structure of PT25-S4-Q23, then, if we had follow PT27-S1-Q23 logic, it gets down to: (negating "most" to "some not", then it follows that some B is not A is ignored, which is what answer choice A says isn't it?

0
User Avatar

Wednesday, Feb 08 2017

51891

PT27.S4.Q25 - all any reporter knows

Hi All,

This is a difficult flaw question and I intent to give a shoot at explaining it, which is different from JY. Please help and check my explanation's validity.

The question link is here: https://classic.7sage.com/lsat_explanations/lsat-27-section-4-question-25/

For flaw question, the first step I do is always noticing the logic and try to draw it out. This helps to distinguish whether the logic fallacy is a formal fallacy or informal fallacy.

Under this question, the logic are breaking down into the following:

Premise 1: Reporter knowledge-->Press Agent-->Tells Everything-->1 reporter knows more->Scoop other reporters

Activator: Tells Everything

Conclusion: Scoop other reporters

By this we notice that it is a SA/NA fallacy.

However, we are not finished. As time consuming as drawing the logic out, the question steam puts the final hurdle.

The question didn't ask "Which one of the following most accurately describes a flaw in the reasoning in the argument", but instead, it says, which one of the following which isn't stated, but is consistent with the flaw.

In other words, we are trying to find one thing that is consistent within the flaw, which we have defined. And we are not defining the flaw here, but a consistent use of language.

Which marks E correct, which translates into: 1 reporter knows more-->Reporter knowledge. I agree, the translation isn't as perfect, but it does draw a great deal of similarity.

Had the question asked to identify the flaw in this question, then B becomes the correct answer, which states: one doesn't have to be a reporter and not scoop the reporters. To see this, refer to PT25-S4-Q23.

Please comment on my explanation.

Thanks,

Panda

0

Hi guys,

I am really confused about this question: https://classic.7sage.com/lsat_explanations/lsat-25-section-4-question-17/

After reading the question, I notice what the author is trying to do. It is trying to argue for a sentimental value to counteract the mayor's argument for the monetary value. And the conclusion argues can be translated as due to the great importance of the sentimental value the item listed should be restored.

From this, I immediately draw up the missing assumption which is almost a principle that is missing: if sentimental value in this case is greater than monetary value then the government needs to fix it despite the fiscal limitation.

As such, I moved into the answer choice and didn't find it or anything close to it.

So I was puzzled between A & B. And finally I chose A for the reason that the argument appealed to emotion. But it is wrong.

Why?

0
User Avatar
51891
Sunday, Jan 15 2017

When reading the question, you should be immediately pick up as of exactly how Young is trying to attack it-by pointing out an uneven distribution.

When I am reading the question there are 2 assumptions that I in my mind:

1) Maybe the number is undistrubuted between the 3 individual

2) Maybe the factory is manufacturing and introduced a different type of problem, and since it has never been seen before, it is unlikely for any quality inspection to pick it up. And furthermore, it sucks that it all landed in the guy's hand.

And when you read the response, it is actually pointing to the first choice. So there is the answer E.

0
User Avatar

Friday, Jan 13 2017

51891

Parallel Reasoning

Hi guys,

I was wondering if you guys have any good tip in terms of approaching parallel questioning? So far, after doing 15 question, the only thing that I came up with:

1) Attention to structure

2) If difficult to understand, supply with an example to fill the referencing words.

Any good tips for this type of questions?

Thanks,

Panda

0
User Avatar
51891
Thursday, Jan 12 2017

@rahelaalam514

Hi Sami,

Thank you for your detailed reply. It had took me days in terms of thinking and at some point it got me into doubting whether the method is really "fool proof" or not. But in either case, I think we have something that we are in agreement of: when the question calls for a detailed analysis of the argument itself, then proper treatment should be supplied.

With that said, I will like to defend the method by pointing out the following:

First, the assumption in itself is a gap that exist between the premise of the argument and the conclusion of the argument. Of course, we can extend this to other circumstance such as when a minor premise is supporting a major premise.

But allow me to demonstrate how this operate by a simple sentence that I encountered yesterday in an email.

"Suggesting that Company Global should buy 4 new equipment due to "excessive pollution seems unreasonable. Your company selected these equipment for the specific application while fully understanding the environment that they would be subject to".

A general sentence, so where is the assumption. And by my method, I will do connection:

Assumption: Due to lack of consideration for environmental damage, the request made by my company is unreasonable.

In other words, the determination factor for the case as reasonable or not is built upon whether the environment which the equipment operated can truly be anticipated.

And to a great extent, an LSAT question is just formulated in such manner but with layers of complexity added on through language cosmetic and contextual information.

With this said, let's take a closer look at another application case.

PT-29-4-20, argument strengthening, except question.

Conclusion: Amphibian (referred as A) is declining in population worldwide.

Major premise: Caused by the declining of depletion of ozone layers.

Minor premise: UV-B damages gens and A's egg lacks UV-B protection

So what is the assumption link that I can make:

Assumption (minor premise with conclusion): The lack of protection and vulnerability to UV-B KILLED the animal.

Assumption 2: (minor premise with conclusion): The gene damage through UV-B exposure is enough to kill the animal.

And of course, since this is causation, the rule of chronology and no vice-versa in terms of relationship also needs to apply in this case.

0
User Avatar
51891
Thursday, Jan 12 2017

Just want to make an add on point about this method.

It is not perfect yet. Yet!

I been spending ours testing it out. And apparently what happens is that every time, every single time, there is that 1 question that will get you. You will presume you know all the sets of items that you are looking for and you mess it up.

0
User Avatar
51891
Thursday, Jan 12 2017

Me either. haha

0
User Avatar
51891
Thursday, Jan 12 2017

Hi There,

so here is my shot at it.

I don't know how long it took you to answer it.

I sort of did it differently. I just jumped in and took the "risk" of connecting them. Of course, along the way by doing this, I am exposing myself to "jumps/assumptions'. But if you had not do this innitialqy, the last sentence should have helped you to chain everything up.

So the answer choice confirms a sufficient/necessary component. And when that happens, the onl y thing that kicks out form this trigger is the NA.

So there you chose E right away and forget the rest of them.

And in regard with your analysis, here are my comment:

Answer choice A: I am not sure if they did no an illegal reversal. But from what I can tell, when you are confirming a NA, nothing checks out. If that is what you meant by illegal reversal.

Rest of them is fine.

The trick behind this question is 3. First, the argument is mixed up where it intended to bother you on your chaining. Second, it is testing about whether you understand if you confirm a NA, nothing kicks. But if you confirms a SA, then NA of the following item kicks.

0

Confirm action

Are you sure?